LSAT and Law School Admissions Forum

Get expert LSAT preparation and law school admissions advice from PowerScore Test Preparation.

 Administrator
PowerScore Staff
  • PowerScore Staff
  • Posts: 8919
  • Joined: Feb 02, 2011
|
#41436
Please post your questions below!
 lathlee
  • Posts: 652
  • Joined: Apr 01, 2016
|
#46179
Hi. I thought A) was the right answer. but this bothers me so much and I have to ask. what is the name of the flaw that is used. I know number and percentage error is involved but i dont think that is the deterministic flaw; i sense there is inappropriate comparison flaw involved.
 Adam Tyson
PowerScore Staff
  • PowerScore Staff
  • Posts: 5153
  • Joined: Apr 14, 2011
|
#62780
I'm not a big fan of naming flaws, lathlee - I don't find that figuring out the name of the flaw is a helpful goal, especially since so many flaws can reasonably be classified under more than one category. Learning the names and descriptions of the most common flaws can help us to create a kind of mental shorthand to help us understand the argument and get to the right answer, but ultimately it is understanding the argument, rather than naming the flaw, that matters, and you seem to understand the flaw here.

I can call this a numbers and percentages flaw, because it fails to consider that "more than another group" is not the same as "more than this group would have experienced otherwise." I can call it an evidence flaw, because evidence about the other group isn't relevant to the group being studied. I can call it a type of relativity flaw, perhaps. Or, I can call it an improper comparison, sort of like an improper analogy, as you did, and I think that's a fine description. Don't worry about what to call it, but focus only on understanding what it is. If you can quickly label it, and if that helps you narrow down the answer choices, that's wonderful, but if you cannot, then don't expend any time or effort struggling to do so. Go for understanding, rather than for labels, and you will move through this faster and with greater confidence.

Good work here! Keep that up!
 g_lawyered
  • Posts: 211
  • Joined: Sep 14, 2020
|
#93433
Hi P.S.,
I thought the argument was casual, in that, the premise is that black belts is the cause and prevent back injuries is the effect. And the conclusion says the opposite, that black belt don't prevent back injuries. For this reason, I thought the flaw was a casual one and the correct answer would the ID the causality or ID 1 of the 3 ways to question the causality (technique used for weaken question): alternative cause/effect, reversal of cause & effect, or coincidence between the 2. Because of my prephrase, I eliminated answer A & B. I narrowed down to C-E.
I eliminated E because it introduced conditional reasoning, which the argument didn't have. And was left with contenders of C & D. I chose C because I thought the language in the answer choice matched language in argument regarding "Likelihood" since the argument states "more likely". As opposed to D, that states "a phenomenon", I didn't think this corresponding with the language in argument.

Can someone please explain why C is incorrect? :-?
Is answer choice D incorrect because of my reasoning?

Thanks in advance!
 Adam Tyson
PowerScore Staff
  • PowerScore Staff
  • Posts: 5153
  • Joined: Apr 14, 2011
|
#93482
I would not classify this as a causal argument, GGIBA003@FIU.EDU, because the author is not using a correlation to prove that one thing causes another. Rather, the author is arguing that one thing is NOT causing another - back belts are NOT causing a reduction in injuries. Thus, you should not look for a causal answer, except perhaps to think about how the data fails to support the conclusion.

Answer C is wrong because the argument does not assume that one factor causally contributes to another, but is arguing the opposite, that one factor is NOT contributing to another.

Answer D is wrong because the author doesn't presume anything about preventing any effect.

Both of these answers are inaccurate descriptions of what occurred in the stimulus. They are, in a word, false, and since Flaw in the Reasoning questions are in the same family as Must Be True questions (which we call the Prove Family), the correct answer must be true. Any answer that describes something that didn't happen, as these do, must be a wrong answer!

Oh, by the way, the use of "a phenomenon" in answer D is not a problem, because the argument does deal with a phenomenon - back injuries in the workplace. That word causes a lot of students to pause and scratch their heads a bit, but it just means "something that happens." Don't let the fancy words confuse you!
 g_lawyered
  • Posts: 211
  • Joined: Sep 14, 2020
|
#93538
Thank you for clarifying that up Adam! I see where I messed the reasoning up. Can you please explain why answer choice A is correct? I didn't recognize 2 groups of people being studied, only the Flegco employees...

Thanks in advance!
 Adam Tyson
PowerScore Staff
  • PowerScore Staff
  • Posts: 5153
  • Joined: Apr 14, 2011
|
#93539
All the people involved are Flegco employees, but there are two sub-groups: those who wear back belts and those who do not. The ones whose work involves heavy lifting wear the belts, but what of the people who do not wear the belts, who must not be doing heavy lifting? The author thinks they are injured less because they do not wear back belts, but perhaps it is because they do not do heavy lifting.
 g_lawyered
  • Posts: 211
  • Joined: Sep 14, 2020
|
#93546
Hi Adam,
The argument is that: back belts are NOT causing a reduction in injuries. So is the method of this question, to point out what the author assumes is incorrect (the assumption is the flaw). I'm trying to make a sense of it because the question stem states: "is MOST vulnerable to criticism to WOTF".
Is the assumption the author makes in answer choice A: that the 2 groups (employees that do wear back belts & employees that don't wear) don't have the same risk factors (heavy lifting) for back injury, is that the flaw?
Am I understanding this correctly?

Thanks in advance!
 Rachael Wilkenfeld
PowerScore Staff
  • PowerScore Staff
  • Posts: 1358
  • Joined: Dec 15, 2011
|
#93562
Right GGIBA003. The author here is treating the all employees at Flegco as having the same amount of risk for back injury. However, we know that all employees are not engaging in the same sorts of activities. Those who wear the braces are engaged in a back muscle heavy occupation. It would make sense that those who do heavy lifting are more at risk for back injuries than those who do not. Answer choice (A) describes that flaw.

Hope that helps.
 g_lawyered
  • Posts: 211
  • Joined: Sep 14, 2020
|
#93564
Thank you Rachael, it does make sense! What's the name of this flaw type? When the author incorrectly assumes to groups are the same when really they are different. I'm trying to make a list of examples of questions that demonstrate this flaw type.

Thanks in advance!

Get the most out of your LSAT Prep Plus subscription.

Analyze and track your performance with our Testing and Analytics Package.